MBE questions with answers/ explanations Flashcards

mixed subjects

You may prefer our related Brainscape-certified flashcards:
1
Q

A recent law school graduate was offered a job as an aide by a state legislator. The legislator told the graduate that before she could begin working, she had to take the following loyalty oath: “I swear to uphold our state and federal Constitutions; to show respect for the state and federal flags; and to oppose the overthrow of the government by violent, illegal, or unconstitutional means.” The graduate told the legislator that the oath is unconstitutional and refused to take the oath.

Is the graduate correct?

A

Yes, as to the promise to respect the flag.

How well did you know this?
1
Not at all
2
3
4
5
Perfectly
2
Q

A landowner and her neighbor owned adjoining tracts of land. No public road abutted the neighbor’s land, so the landowner granted the neighbor an express easement over the north 25 feet of the landowner’s land. However, the following month the county extended the public road to the neighbor’s land, and he ceased using the easement for ingress and egress.

Twenty years later, the neighbor conveyed the easement to his friend, who owned the land adjoining the other side of the landowner. The following year, the neighbor conveyed his land to the landowner. None of the parties has used the easement since the public road was extended. The jurisdiction has a 15-year statute of limitations for acquiring property interests by adverse possession.

At what point was the easement extinguished?

A

When the neighbor conveyed his land to the landowner.

The easement was extinguished when the neighbor conveyed his land to the landowner. An easement is extinguished when the easement is conveyed to the owner of the servient tenement. For an easement to exist, the ownership of the easement and the servient tenement must be in different persons. (By definition, an easement is the right to use the land of another for a special purpose.) If ownership of the two property interests comes together in one person, the easement is extinguished.

How well did you know this?
1
Not at all
2
3
4
5
Perfectly
3
Q

Congress passed legislation banning the hunting of snipe birds within the United States. The range of the snipe is quite limited; they are found primarily in only one state, although they migrate annually to several nearby states. Hunters from throughout the United States have traditionally traveled to the snipe’s home state during snipe hunting season, bringing considerable revenue into the state. A state statute allows hunting of snipe during a two-week period in November and charges a $50 license fee for state residents and a $250 fee for hunters from other states. The bag limit is one snipe bird per licensed hunter.

Is the state statute allowing snipe hunting valid?

A

No, because of the Supremacy Clause.

The state statute is invalid because of the Supremacy Clause. Under the Clause, if the federal government adopts legislation that it has the power to adopt, the federal legislation is supreme, and a conflicting state law is rendered invalid. The federal law here, banning the hunting of snipe, is within the federal government’s power under the Commerce Clause, which gives the government power to regulate anything that might affect interstate commerce.

How well did you know this?
1
Not at all
2
3
4
5
Perfectly
4
Q

A developer subdivided a 25-acre tract of land into 100 quarter-acre lots. On each lot she built a two-unit townhouse. The deeds to each of the purchasers contained a covenant that “the grantee, his heirs and assigns” would use the property only for single-family use. All deeds were promptly and properly recorded. Subsequently, the zoning laws were amended to allow multifamily use within the subdivision. Six months later, a social worker offered to purchase an original owner’s unit that was for sale. The social worker informed the owner that she planned to operate a halfway house out of the unit, an activity in conformity with the applicable zoning regulations. Therefore, the owner did not include the single-family restriction in the deed to the social worker.

If a neighbor, who purchased his lot from the developer, seeks to enjoin the operation of the halfway house, will he succeed?

A

Yes, because the social worker had notice of the restrictive covenant.

The neighbor will succeed in enjoining the operation of the halfway house because the social worker had notice of the restrictive covenant. A covenant runs with the land to a subsequent purchaser with notice of the covenant if it touches and concerns the land and is intended to run. Notice may be actual or constructive. Here, the social worker was on record notice of the covenant because the original owner’s deed was recorded. Restricting land to single-family use touches and concerns the land, and it is evident that the developer and the original owners, including the neighbor, intended it to run with the land by use of the language “grantee, his heirs and assigns.” The social worker thus will be bound even though her deed did not refer to the covenant.

How well did you know this?
1
Not at all
2
3
4
5
Perfectly
5
Q

A town adopted an ordinance providing that a person must have been a resident of the town for at least one year to be eligible to vote in school board elections. A resident who moved to the town seven months ago attempted to register to vote in the school board elections scheduled for the next month. However, the town clerk refused to register the resident because he will not have resided in the town for a full year prior to the election. The resident filed a class action suit on behalf of all of the new residents of the town, challenging the validity of the one-year residency requirement.

Which of the following statements is correct?

A

The resident will prevail even if the matter is not decided until after next month’s election.

The resident will prevail even if the matter is not decided until after the election, because the suit is not moot and the residency requirement is unconstitutional.

How well did you know this?
1
Not at all
2
3
4
5
Perfectly
6
Q

The defendant was on trial for murdering his mother, who was found dead in her bathtub. At trial, the prosecutor called the nurse of the defendant’s aunt to testify to what the aunt told the nurse just before the aunt died of cancer. The nurse is prepared to testify that, shortly before she died, the aunt stated, “I know I don’t have much longer to live, so I must tell someone what my nephew said to me yesterday. He told me that he was very angry with his mother and that he wanted to kill her and make it look like an accident!”

Should this testimony be admitted?

A

No, because it is inadmissible hearsay.

How well did you know this?
1
Not at all
2
3
4
5
Perfectly
7
Q

A cattle-producing state adopted a statute requiring any food service business operating in the state to serve beef raised in the United States. A licensed hot dog vendor who worked at a football field within the state and who had been buying hot dogs made with foreign beef for the past several years estimated that switching to an all-beef hot dog made from United States beef would reduce his profits by 10%. An attorney hired by the vendor to challenge the statute discovered during research into the case that most of the footballs used at the football field at which the vendor worked were made of foreign leather.

Which of the following grounds is the vendor’s best argument against the constitutionality of the state statute?

A

The statute burdens foreign commerce.

How well did you know this?
1
Not at all
2
3
4
5
Perfectly
8
Q

A new federal law prohibited the use of various pesticides in areas with a certain population density near navigable waters. A city located in the southeastern United States was plagued by a sharp increase in disease-carrying mosquitoes. The city’s board of health recommended that all residential areas be sprayed with a pesticide proven to be highly effective against mosquitoes. Despite the fact that the federal law would prohibit use of that pesticide in these areas, the city council passed an ordinance adopting the board of health plan, relying on the opinions of several independent experts that the health benefits of reducing the mosquito population outweighed the risks of spraying. An environmentally minded citizen of the city brought an action in federal court challenging the ordinance.

Assuming that the citizen has standing, is the court likely to find the ordinance valid?

A

No, because it conflicts with a federal law that Congress had the power to make under the Commerce Clause.

Congress’s power to regulate commerce has been construed broadly, so that it may regulate any activity, local or interstate, that either in itself or in combination with other activities has a substantial economic effect on interstate commerce. If Congress has determined that the use of chemical pesticides and their runoff into waterways (which are channels of interstate commerce) will have an overall detrimental impact on the environment, this determination will be sufficient in this case to satisfy the standards established by the Supreme Court. Therefore, the law probably is a valid exercise of the commerce power. Any state or local action that conflicts with a valid act of Congress is invalid under the Supremacy Clause.

How well did you know this?
1
Not at all
2
3
4
5
Perfectly
9
Q

The state has the following homicide statutes:

Murder is the unlawful killing of a human being with malice aforethought. Such malice may be express or implied. It is express when there is manifested a deliberate intention to unlawfully take away the life of a fellow creature. It is implied when no considerable provocation appears or when the circumstances attending the killing show an abandoned and malignant heart. All murder that is perpetrated by willful, deliberate, or premeditated killing or committed in the perpetration of or attempt to perpetrate arson, rape, robbery, or burglary is murder of the first degree. All other kinds of murders are of the second degree.
The defendant and her associate entered a jewelry store to shoplift a diamond bracelet. Just as the defendant put the bracelet into her pocket, a sales clerk saw her and grabbed her by the wrist. The associate grabbed a knife from one of the silver displays and lunged at the sales clerk, but then a store guard shot and killed her. The defendant is charged with the first degree murder of her associate.

Which of the following is the defendant’s strongest argument?

A

The defendant cannot be convicted of murder because the associate’s death was not murder but justifiable homicide

The defendant’s strongest argument is that her associate’s death was justifiable homicide. Most courts today would not allow the defendant to be convicted on a felony murder theory when a co-felon is killed by a third party during the crime. Some courts base this result on the fact that the person who did the killing was justified in doing so.

How well did you know this?
1
Not at all
2
3
4
5
Perfectly
10
Q

An environmentalist divided her 25-acre property into 100 quarter-acre residential lots. At the time the environmentalist sold her lots, there was a recycling center about one mile from the western boundary of the development. She included in the deed of all 100 grantees the following provision:

“Grantee covenants for herself and her heirs and assigns that all aluminum cans, glass bottles, and grass clippings of Grantee and her heirs and assigns shall be recycled. This covenant runs with the land and shall remain in effect as long as there is a recycling center within five statute miles of the development.”

A buyer purchased a lot in the development. Her deed, which contained the recycling clause, was duly recorded. Two years later, the buyer decided to give the property to her niece as a gift. The niece’s deed to the property contained the recycling covenant, and she too recorded her deed. Shortly after the niece took possession of the house, the recycling center moved its location to a new site about four and a half miles from the development. When the niece put the house up for sale, she said nothing to prospective buyers about recycling.

The house was purchased by a veteran who had lost the use of his legs. The veteran’s deed did not contain the recycling clause, and he hired a local disposal service to carry away his garbage and a landscaper to maintain the yard. The landscaper bagged the grass clippings and they were removed by the disposal service, which put all the trash and clippings in a landfill. When the veteran’s neighbors informed him of his duty to recycle, he told them that he knew nothing of the covenant and that it would be difficult for a person in his physical condition to haul cans, bottles, and clippings to the recycling center. Unfazed, the neighbors filed suit to require the veteran to comply with the covenant or pay damages.

The veteran’s best defense is which of the following?

A

The covenant does not touch and concern the land.

he veteran’s best defense is that the covenant does not clearly “touch and concern” the land. While recycling may benefit the community at large, “touch and concern” involves the relationship between landowners at law. Recycling by the veteran does not directly benefit the other landowners in the use and enjoyment of their land.

How well did you know this?
1
Not at all
2
3
4
5
Perfectly
11
Q

Which of the following suits would not fall within the United States Supreme Court’s original jurisdiction under Article III, Section 2?

A

A suit by a state seeking to assert the interest of its citizens in retaining diplomatic relations with a foreign nation.

A suit by a state seeking to assert the interest of its citizens in retaining diplomatic relations with a foreign nation is not within the Supreme Court’s original jurisdiction. Under Article III, Section 2, the United States Supreme Court has original jurisdiction in all cases affecting ambassadors, other public ministers, and consuls, and in which a state is a party.

How well did you know this?
1
Not at all
2
3
4
5
Perfectly
12
Q

A cattle-producing state adopted a statute requiring any food service business operating in the state to serve beef raised in the United States. A licensed hot dog vendor who worked at a football field within the state and who had been buying hot dogs made with foreign beef for the past several years estimated that switching to an all-beef hot dog made from United States beef would reduce his profits by 10%. An attorney hired by the vendor to challenge the statute discovered during research into the case that most of the footballs used at the football field at which the vendor worked were made of foreign leather.

Which of the following grounds is the vendor’s best argument against the constitutionality of the state statute?

A

The statute burdens foreign commerce.

The best argument against the constitutionality of the state statute is that it burdens foreign commerce. For all practical purposes, the power to regulate foreign commerce lies exclusively with Congress. Therefore, a state that adopts legislation requiring private vendors to favor United States products over foreign products, as the state did here, may be acting outside the scope of its powers.

How well did you know this?
1
Not at all
2
3
4
5
Perfectly
13
Q

While cross-examining a defendant on trial for robbery and assault with a deadly weapon, the prosecutor asks him whether he was convicted of fraud within the previous year.

Is this question proper?

A

Yes, because it tends to show that the defendant would lie.

How well did you know this?
1
Not at all
2
3
4
5
Perfectly
14
Q

A landlord leased office space to a business owner for five years, ending on November 1, reserving a yearly rent of $24,000, payable monthly. On October 1 of the fifth year, the business owner notified the landlord that he was preparing to move, but would greatly appreciate if the landlord could extend the lease for a month or two. On October 10, the landlord wrote to the business owner that she thought they could reach a satisfactory arrangement, but did not hear back from the business owner. The business owner did not vacate the office until November 20. On November 30, the landlord received a check from the business owner in the amount of $1,333 for “November’s rent” and a note that he had vacated the premises.

If the landlord brings an action against the business owner for additional rent, how will the court rule?

A

The business owner is bound to a year-to-year tenancy, because he did not vacate the premises until November 20.

The court will rule that the business owner is bound to a year-to-year tenancy because he is a hold-over tenant. When a tenant fails to vacate the premises after the termination of his right to possession, the landlord may: (i) treat the hold-over tenant as a trespasser and evict him; or (ii) bind the tenant to a new periodic tenancy. The terms and conditions of the expired tenancy apply to the new tenancy. At least in commercial leases, the new tenancy will be year-to-year if the original lease term was for one year or more. Here, the businessman was a tenant for years because his lease was for a five-year fixed period of time. A tenancy for years ends automatically on its termination date. Therefore, as of November 1, the business owner became a hold-over tenant and the landlord had a right to bind him to a new periodic tenancy. Because the original lease was for more than one year, the business owner may be held to a year-to-year tenancy, at the stipulated rent of $24,000 per year.

How well did you know this?
1
Not at all
2
3
4
5
Perfectly
15
Q

An antique lover spotted a beautiful Early American bedroom ensemble at her favorite antique store. The ensemble included a bed, a mirror, and two dressers. Over a period of several weeks, the shop owner and the antique lover negotiated over a price, but they were unable to come to an agreement.

On April 3, the shop owner and the antique lover signed a statement whereby the shop owner offered to sell to the antique lover an Early American bedroom ensemble, recorded as items 20465, 20466, 20467, and 20468 in the shop’s registry, if the parties agree upon a price on or before April 12.

On April 6, the shop owner sent a letter to the antique lover, telling her that she could have the bedroom ensemble for $22,000. Also on April 6, the antique lover sent a letter to the shop owner telling him that she was willing to pay him $22,000 for the bedroom ensemble. Both parties received their letters on April 7.

Without assuming any additional facts, which of the following statements is most correct as of April 8?

A

No contract exists between the shop owner and the antique lover, because of a lack of mutual assent.

THERE WAS NO acceptance

How well did you know this?
1
Not at all
2
3
4
5
Perfectly
16
Q

A union filed suit against a corporation, known for its antiunion management, asserting that its members were being discharged in retaliation for membership in the union rather than for any failure to perform their jobs properly. Under the pretrial discovery orders, a union employee was allowed to examine all of the records held in the corporation’s files concerning discharge of employees for a seven-year period prior to the instigation of suit by the union. The employee sorted through this large volume of material and discovered that persons who were union activists usually had “lack of corporate spirit” listed as their reason for discharge, while other fired workers tended to have more specific grounds for discharge listed, e.g., persistent lateness. The employee developed a chart showing grounds for dismissal of union members versus nonmembers based on the data in the files. At the trial, the union placed the employee on the stand. She testified in some detail regarding how she had conducted her research. The employee brought out the chart and the union’s lawyer asked that the chart be admitted into evidence. The corporation’s attorney objected.

How should the court rule on the admissibility of the chart?

A

Admissible, because copies of the original documents upon which the chart was based were available to the corporation prior to trial.

How well did you know this?
1
Not at all
2
3
4
5
Perfectly
17
Q

A defendant was charged with arson (a felony) of an antique shop. Only one corner of the shop was damaged before the fire was extinguished. Under a plea agreement, the defendant pled guilty and received a suspended sentence. Because the owner of the shop had not yet insured a recently acquired 400-year-old refectory table that was destroyed by the fire, he sued the defendant for damages. At trial, the owner offers the properly authenticated record of the defendant’s conviction for arson.

Should the record be admitted into evidence?

A

Yes, as proof that the defendant set the fire.

How well did you know this?
1
Not at all
2
3
4
5
Perfectly
18
Q

The owner of a corner lot allowed a hedge on his property to become overgrown, obstructing the view of motorists at that corner. Two motorists were driving inattentively and each ran a stop sign at the intersection bordering the lot. Their cars collided in the intersection and one of the motorists was injured. She sued the owner of the lot. The jury determined that the lot owner was 10% at fault and each of the motorists was 45% at fault.

Will the injured motorist recover damages from the lot owner?

A

Yes, because it was foreseeable that motorists could be injured if the hedge was not cut back.

How well did you know this?
1
Not at all
2
3
4
5
Perfectly
19
Q

A critically acclaimed movie that had received a number of awards opened in a small town. The film had portrayals of nudity and scenes involving sexuality, but its advertising was very tasteful and concentrated on its critical acclaim and its receipt of seven Academy Award nominations. Nevertheless, when the movie opened in the small town, there was a public outcry against it, including picketing. The town, which had been founded in the late nineteenth century by a fundamentalist religious group, remained very conservative and highly religious, and was the only community in the state where a consensus of the community would find the movie to be obscene. The town prosecutor went to the local court seeking an injunction to halt the showing of the movie. The theater owner refused to voluntarily stop showing the film and appeared in court to defend against the proposed injunction.

What is the owner’s best defense?

A

The film has proven artistic merit.

The theater owner’s best defense is that the film has proven artistic merit. The First Amendment generally protects the right of freedom of speech, and this freedom includes the right to show movies. Thus, to enjoin the showing of the movie here, the city will have to prove that the speech involved is unprotected speech. Obscenity is the category of unprotected speech most relevant here. The Court has defined obscenity as a depiction of sexual conduct that, taken as a whole, by the average person, using contemporary community standards: (i) appeals to the prurient interest in sex; (ii) portrays sex in a patently offensive way; and (iii) using a national, reasonable person standard, does not have serious literary, artistic, political, or scientific value. If the theater owner shows that the film has proven artistic merit, it cannot be held to be obscene because the third element of the above definition will have failed.

How well did you know this?
1
Not at all
2
3
4
5
Perfectly
20
Q

An airline passenger nearly killed in a crash is suing the airline for personal injuries. To prove the extent of his injuries, the passenger offers a videotape taken by a local news station immediately after the crash that shows serious burns covering much of the passenger’s face. The airline moves to exclude the videotape on grounds that its probative value is substantially outweighed by the danger of unfair prejudice.

In making his ruling, which of the following is NOT appropriate for the judge to consider?

A

The videotape will make it more likely that the passenger will win the suit.

How well did you know this?
1
Not at all
2
3
4
5
Perfectly
21
Q

A defendant is being prosecuted in federal court for illegally transporting persons across state lines for immoral purposes. The prosecutor alleges that her route was from New York to Tampa. The court takes judicial notice of the fact that it is impossible to get from New York to Tampa without crossing a state line.

What is the effect of the court’s action?

Responses

A

The prosecutor’s burden of producing evidence on this point is satisfied.

How well did you know this?
1
Not at all
2
3
4
5
Perfectly
22
Q

A defendant charged with murder admitted to the killing but claimed that he shot the victim in self-defense as she attacked him with a knife. At trial, the investigating officer testified about the scene of the crime and the condition of the victim at the time of death. The prosecutor showed the officer a photograph of the scene of the crime taken by the police photographer and asked the officer whether the photograph accurately depicted what he had observed at the scene. The officer testified that it did. The photograph showed the deceased lying in a pool of blood with both her hands cut off.

Should the court admit the photograph?

A

Yes, unless the court determines that its probative value is substantially outweighed by the danger of unfair prejudice.

How well did you know this?
1
Not at all
2
3
4
5
Perfectly
23
Q

In a property dispute, a granddaughter claims that her grandfather gave her a deed to his home just before he died. The grandfather’s son claims that the property is rightfully his by a previously executed will. At issue is the authenticity and content of the deed. The granddaughter begins to testify as to the content of the deed, but the son’s attorney objects.

Should the court sustain the objection?

A

No, if the judge is satisfied that the deed could not be found after a reasonable search.

How well did you know this?
1
Not at all
2
3
4
5
Perfectly
24
Q

A defendant is on trial for first degree murder and the prosecution wants to introduce a recorded telephone call by the victim to the police just before she was killed. Distraught, the victim failed to identify herself during the call. A witness is called to testify that the voice recorded was that of the victim’s.

Under which of the following circumstances would the trial court most likely sustain an appropriate objection by the defense counsel to admission of the tape recording into evidence?

A

The witness had heard the victim’s voice in several recorded telephone conversations between the victim and the victim’s father, and the victim’s father had told the witness that it was the victim with whom he was speaking.

How well did you know this?
1
Not at all
2
3
4
5
Perfectly
25
Q

At the defendant’s trial for a brutal assault, in which the victim identified the defendant as her assailant, the defense calls a witness who will testify that he has worked with the defendant for 20 years and that all of his business associates regard the defendant as an honest person.

If the prosecution objects to the witness’s testimony, for which party should the court rule?

A

The prosecution, because the testimony presents no evidence of any relevant character trait.

How well did you know this?
1
Not at all
2
3
4
5
Perfectly
26
Q

The defendant, a used car seller, is on trial for criminal fraud, charged with selling used cars with major mechanical problems while representing to buyers that the cars were mechanically sound. The defendant claims that she had no knowledge the cars were not fit for sale. At trial, the prosecution offers evidence to show that, eight months prior, the defendant was fired from a different used car lot for knowingly selling defective automobiles with major mechanical problems.

What is the best basis for admitting this evidence?

A

As evidence of the defendant’s criminal intent.

How well did you know this?
1
Not at all
2
3
4
5
Perfectly
27
Q

Evidence Workshop
12 of 2512 of 25 Items

Question
A defendant on trial for robbery took the stand in his own defense and testified that the robber was his neighbor. The jury acquitted the defendant based on this testimony. The neighbor was then indicted and brought to trial for the robbery. At that trial the prosecution called the defendant from the first trial to the stand, expecting him to incriminate the neighbor. Surprisingly, the defendant testified: “My neighbor didn’t have anything to do with that robbery, but I know who did! I committed the robbery myself.” When asked about the testimony he gave at his own trial, the defendant insisted he didn’t remember anything about it.

Finding her case in shambles, the prosecutor calls a juror from the first trial to the stand as a witness, who is prepared to testify that the defendant said at the first trial that the neighbor committed the robbery.

On objection by the defense, should the court admit the juror’s testimony

A

Yes, to impeach the first defendant’s credibility as a witness, and as substantive evidence of the neighbor’s guilt.

How well did you know this?
1
Not at all
2
3
4
5
Perfectly
28
Q

A defendant is on trial for a murder that occurred during a robbery at the victim’s home. A witness helped the police artist compose an accurate depiction of the defendant. The witness was unavailable at the time of trial and the prosecutor offers the sketch into evidence.

Is the sketch admissible?

A

No, as hearsay not within any exception.

How well did you know this?
1
Not at all
2
3
4
5
Perfectly
29
Q

At a trial in which a pedestrian is suing a driver, a hospital record was admitted into evidence that included the following statement: “The pedestrian’s leg was run over by a car driven by a driver who blew through a red light while the pedestrian was crossing in a crosswalk.”

The driver’s attorney now wishes to admit the other portion of the hospital record, which says, “The pedestrian stepped off the curb without first looking both ways for traffic.”

How should the court rule?

A

Admit the statement on fairness grounds because the plaintiff has the other portion of the record.

How well did you know this?
1
Not at all
2
3
4
5
Perfectly
30
Q

The son of a famous author who has not been seen in two years brings an action against an insurance company to compel payment of the proceeds of the author’s insurance policy, for which the son is the sole beneficiary.

The son introduced evidence that, on the day the author disappeared, a plane left from the city where she lived and was lost while traveling over the ocean. The manifest of the airline was introduced showing that a passenger with a name similar to hers was aboard the airliner. The son wants to testify that his mother told him that she was going to be on that plane and, to preserve her privacy, was going to travel under the name that matches the name in the manifest.

Is the son’s testimony admissible?

A

Yes, because it is a relevant indication of state of mind.

How well did you know this?
1
Not at all
2
3
4
5
Perfectly
31
Q

A camper sued the manufacturer of thermal underwear, alleging that while he was attempting to stomp out a fire, the camper’s underwear caught fire and burned in a melting fashion up to his waist because it was defective, and that, a half hour later, he suffered a heart attack as a result of the burns he suffered.

A physician hearing the camper testify to the events that occurred is called by the camper and asked whether the camper’s heart attack could have resulted from the burns.

Is his opinion admissible?hjgfc

A

Yes, as a response to a hypothetical question.

How well did you know this?
1
Not at all
2
3
4
5
Perfectly
32
Q

A landowner validly conveyed a small office building to the Green Party “as long as they use it for operating quarters until the next presidential election.” After the next presidential election, which was in three years, the building would go to a private organization that monitors and prepares comprehensive listings of gas prices throughout the country. A year after the conveyance, the landowner died, validly devising all of her property to her son. Although this jurisdiction is a common law jurisdiction with respect to all real property considerations, the state’s probate laws provide that future interests or estates in real property may be passed by will or descent in the same manner as present or possessory interests. Last week, the Green Party and the gas monitoring organization joined together to sell the office building in fee simple absolute to a developer. The son filed suit to prevent the sale of the property to the developer.

In this action, who should prevail?

A

The son, because he did not sign the contract of sale.

How well did you know this?
1
Not at all
2
3
4
5
Perfectly
33
Q

Several members of a small terrorist group are on trial in federal court for conspiring to bomb a military installation. The prosecution would like to introduce the testimony of a military guard at one of the installation’s gates. The guard had been present when a bomb that was being planted by a member of the group had exploded prematurely. The guard will testify that she ran over to administer first aid to the member, who in great pain told her that his group was in the process of planting three other bombs in other areas of the military installation and was going to detonate them all at the same time to get publicity for their cause. The guard will also testify that the member disclosed the locations of the other bombs and the names of two other members of the group. The authorities were able to prevent the other bombings and arrest the other members of the group. The member died from his injuries.

What is the best basis for allowing the guard to testify as to the member’s statements?

A

As a statement against interest.

How well did you know this?
1
Not at all
2
3
4
5
Perfectly
34
Q

A minor league ballplayer hit a fly ball over the wall and out of the park during a game and struck a woman riding along the adjacent street on a bicycle. The woman sued the ballplayer for negligence. The woman alleged that the ballplayer had often hit balls out of the park and was aware that he had previously struck a car driving down the street.
Assuming the woman’s allegations are correct, is she likely to prevail?

A

No, because the ballplayer did not breach a duty of care towards the woman.

How well did you know this?
1
Not at all
2
3
4
5
Perfectly
35
Q

A plumber working for a company providing plumbing services to commercial and industrial establishments was required to be “on call” for emergency plumbing services 24 hours a day, and was required to drive his company van home each night so he would have all of his tools and equipment at hand for any calls. However, he was not permitted to use the company van for personal errands. On his way home one afternoon, he took a detour toward a supermarket a few blocks away to pick up some items for dinner. While entering the supermarket parking lot, he drove negligently and struck a pedestrian, seriously injuring him. The pedestrian filed suit against the plumber’s company in a jurisdiction that maintains traditional common law rules regarding contribution and indemnity, and the jury awarded him $100,000 in damages, which the company paid.

If the company sues the plumber to recoup its loss in the lawsuit, which party will prevail?

A

The company can recover 100% of the judgment as an indemnity, because the plumber was negligent, not the company.

How well did you know this?
1
Not at all
2
3
4
5
Perfectly
36
Q

An automotive engineer announced that he had developed a carburetor that will enable cars to achieve 100 miles per gallon of fuel, and that he will allow the carburetor to be inspected next month. Soon after, a former employer of the engineer brought an action to prohibit the engineer from displaying the carburetor, claiming that the engineer probably had stolen the carburetor’s design from the employer. The court granted the employer a temporary restraining order prohibiting the engineer from disclosing any mechanical details of his carburetor, and ordered a hearing to be held in one week to determine whether a preliminary injunction should be issued. Because each party would have to reveal the mechanical details of his designs at the hearing, the employer requested that the hearing be closed to the public and that the record be sealed to avoid revelation of his designs. The court granted the request. A reporter for a monthly automobile magazine heard about the case and wanted to attend the hearing. When he was told that the hearing would be closed, he filed an action to have it opened.

What is the reporter’s best argument for opening the hearing?

A

Closure is not necessary to preserve an overriding interest here.

How well did you know this?
1
Not at all
2
3
4
5
Perfectly
37
Q

A plaintiff sued a defendant for damages suffered when a load of bricks fell off the defendant’s truck directly in front of the plaintiff while she was driving on a highway. The plaintiff charged that the defendant was negligent in supplying his truck with a defective load chain clamp, which helped tie the load to the bed of the truck, and in failing to secure the load properly on the truck. The plaintiff calls a witness who testifies that he was formerly employed as a truck driver and is an acquaintance of the defendant. The witness further testifies that immediately prior to the accident he had coffee with the defendant at a cafe, and mentioned to the defendant that the tie chains holding the load of bricks looked kind of loose.

Assuming proper objection by the defendant’s attorney, how should the court rule on the admissibility of such testimony?

A

Admissible nonhearsay.

How well did you know this?
1
Not at all
2
3
4
5
Perfectly
38
Q

A woman was struck by a brick with her name scrawled on it that was thrown through her bedroom window. The victim believes that her ex-boyfriend, who is a gang member, threw the brick because she has become active in anti-gang groups, but she did not actually see him throw it. The ex-boyfriend is arrested and put on trial for battery.

Assuming that all notice requirements have been met, which of the following items of the victim’s proposed testimony is LEAST likely to be admitted?

A

On another occasion, the victim had seen her ex-boyfriend throw a rock through the window of a rival street gang member.

How well did you know this?
1
Not at all
2
3
4
5
Perfectly
39
Q

A homeowner, a citizen of State A, hired an electrician, a citizen of State B, to fix the wiring in her basement and hired a gas worker, also a citizen of State B, to install a new gas stove in her kitchen. Unfortunately, the home caught fire and burned down while they were both working on their separate jobs. The homeowner sued the gas worker for negligence in federal court in State A, seeking $100,000. The homeowner promptly served the gas worker, and the gas worker timely filed an answer with the court. One month after filing the answer, the gas worker moved to file and serve a third-party complaint against the electrician, alleging that the electrician was the sole cause of the accident.

Which of the following arguments is most likely to achieve the electrician’s goal of dismissal of the third-party complaint?

A

The gas worker’s claim against the electrician is not a proper third-party claim.

The electrician’s best argument is that the gas worker’s claim against the electrician is not a proper third-party claim. Under Rule 14, a defendant may assert a third-party claim against “a nonparty who is or may be liable to it for all or part of the claim against it.” In other words, a third-party claim must be a derivative claim; the third-party plaintiff must be seeking indemnification or contribution from the third-party defendant. Here, the gas worker’s claim is not that the electrician must indemnify him or that the electrician is a joint tortfeasor who may be jointly liable under principles of contribution. Rather, the gas worker is alleging that he (the gas worker) is not liable and that the electrician is. Because the claim is not derivative, it is not properly asserted as a third-party claim under Rule 14.

How well did you know this?
1
Not at all
2
3
4
5
Perfectly
40
Q

A landowner included in his will a provision giving “all of my property, both real and personal, wherever situated, to my widow for life, and after her death to any of our children who may survive her.”

What is the gift to the children?

A

A contingent remainder.

How well did you know this?
1
Not at all
2
3
4
5
Perfectly
41
Q

A hockey player who was playing in the final game of the season before a hostile crowd in the opponent’s packed stadium had an opportunity to get his team into the playoffs, but he missed a shot into an open net as the horn sounded, ending the game. As the crowd cheered and jeered, the puck bounced back to him and he shot it in anger toward the stands. A fan who had been looking the other way turned back toward the rink just in time to be struck in the face by the puck. He suffered a broken nose and a severe gash under his eye. After the game, the league commissioner fined the player for violating league rules by intentionally directing the puck out of the playing area.

If the fan sues the player for battery, will the fan likely prevail?

A

Yes, because the player knew that it was substantially certain that a fan would be hit by the puck.

42
Q

The plaintiff is suing the defendant for personal injuries suffered when she was struck by the defendant’s car. The plaintiff testified regarding the extent of her injuries. The defendant seeks to testify that when she visited the plaintiff in the hospital, the plaintiff said that if the defendant paid all of her medical bills and gave her $25,000, she would not institute legal proceedings. The plaintiff’s attorney objects to the offered testimony.

How should the judge rule on the admissibility of the testimony?

A

Inadmissible, because it was an offer of compromise.

43
Q

A beneficiary has filed a petition in the probate court to contest the validity of a testator’s will. The beneficiary contends that when the testator executed the will eight years before, he had a severe mental illness and was incapable of forming a valid testamentary intent. In support of this contention, the beneficiary seeks to offer an affidavit prepared by the testator’s former attorney, which states that she was asked to prepare a will for the testator just four months before this will was made. The attorney had refused to do so because it was her opinion that the testator seemed incoherent and paranoid.

How should the judge rule on the admissibility of this affidavit?

A

Inadmissible, because it is hearsay not within any exception.

44
Q

A dog owner lived next door to a day care center. Because he had a large yard and there were no applicable zoning restrictions, he installed a kennel and began training attack dogs to sell to businesses. As soon as he opened the business and posted signs in front advertising the exceptional ferocity of the dogs, some parents who had children enrolled in the day care center became alarmed at the prospect of the dogs right next to the yard where the children played, especially because the children could see and hear the dogs being taught to attack people. Within a few months of the dogs’ arrival next door, the owner of the day care lost 10% of her enrollment.

If the day care owner brings a nuisance action against the dog owner, what will be the most critical factual issue that the trier of fact must resolve to determine who should prevail?

A

Whether the day care owner’s use of her property makes her business abnormally sensitive to the presence of the dogs.

45
Q

Based on recommendations of a state commission studying the effect of pornographic films on violent criminal activity, a state adopted legislation banning films intended for commercial distribution that appealed as a whole to the prurient interest in sex of the average person in the community, portrayed sex in a patently offensive way to citizens of the state, and which a reasonable person in the United States would find had no serious literary, artistic, political, or scientific value.

In ruling on a constitutional challenge to the legislation from a film distributor in the state who was convicted of distributing films in violation of the legislation, will the federal court likely find the legislation to be constitutional?

A

Yes, because it uses a national “reasonable person” standard for determining the social value of the work.

46
Q

A foreign student who had entered the United States on a student visa four years ago was notified by federal immigration authorities that he was subject to being deported because his visa had expired. Federal law provided that an alien who is subject to being deported has the right to appear before an administrative officer appointed by the Attorney General’s office for a hearing on whether he should be deported. This officer, appointed by the executive branch of the government, has the right under law to make a final order concerning whether the alien should be deported. After a hearing, the administrative officer entered an order allowing the student to remain in the United States as a permanent resident.

However, a congressional rule permitted the House of Representatives, by resolution, to deport “undesirable aliens.” After the administrative judge entered his order, the House passed a resolution that the student should be deported. The student petitioned the federal court to declare the legislative resolution invalid.

Should the court find the resolution to be valid?

A

No, because the federal law removed congressional power with regard to aliens in this circumstance, and the resolution of the House violates the separation of powers doctrine.

47
Q

A car owner lent her automobile to her girlfriend for the specific purpose of picking up a pizza that the owner and the girlfriend had ordered for dinner. The girlfriend drove to the shopping mall where the pizzeria was located and parked the owner’s car there. Instead of going directly to the pizzeria, the girlfriend went into a bookstore, browsed, and eventually purchased a book. The girlfriend then went to the pizzeria and picked up the pizza, which had been ready for 15 minutes. Just as the girlfriend left the pizzeria to return to the car, another car struck the owner’s parked car, causing extensive damage to the car. The owner did not carry collision insurance, and the car required $800 worth of body work.

If the owner sues the girlfriend on a negligence theory for damage to the car, who will prevail?

A

The girlfriend, because she did not create a foreseeable risk of damage to the owner’s car.

48
Q

A man who belonged to an ancient religion whose rituals require the use of bald eagle feathers traveled to an area where bald eagles were known to roost. After searching the area, he found a fallen eagle feather and returned home. A few weeks later, the man showed the feather to an acquaintance, who happened to be a state park ranger, and explained how the feather was obtained. The ranger informed the man that a state anti-poaching law makes any possession of a bald eagle feather without a special permit a crime. The ranger then cited the man for possession of the feather and confiscated it.

At the man’s trial for violating the state bald eagle feather possession statute, which of the following constitutional arguments is most appropriate for the prosecution to make?

A

The statute is a neutral law that only incidentally burdens the man’s rights under the First Amendment.

49
Q

A father was angry at his son’s coach because the coach would never let the son into a game. In order to exact revenge, the father decided to plant an incendiary device on the coach’s front porch. The father believed the device would start a fire that would destroy the coach’s home and perhaps injure him as well. However, the father made a mistake while assembling the incendiary device, and it was impossible for the device to do any harm. When the device went off, it did nothing more than produce a foul odor.

If the father is charged with attempted murder and attempted arson in a common law jurisdiction, which of the following decisions is most likely to be reached by the court?

A

The father is not guilty of attempted murder, but he is guilty of attempted arson.

50
Q

A plaintiff is suing a defendant in federal court for personal injuries arising out of an automobile accident. The defendant’s automobile insurance policy covers the accident.

Must the defendant disclose the existence of the insurance policy

A

Yes, because it is required under the Federal Rules.

51
Q

Congress enacted a statute that purported to ban all discrimination against African-Americans in any commercial transaction taking place within the United States.

Would the statute most likely be held constitutional?

A

Yes, under Thirteenth Amendment provisions barring badges or incidents of slavery.

52
Q

A hardware store ordered 200 cans of wood stain in various shades. The written contract between the store and manufacturer provided that 100 cans of stain would be delivered on April 30, and the remaining 100 cans would be delivered on June 30. Payment would be due at the time of each delivery. The first shipment arrived on April 30. Sales of the stain were brisk, but 25 customers almost immediately returned their stain, complaining that it was not the color indicated on the can. The store owner called the manufacturer and informed it of the problem. The manufacturer truthfully told the owner that they had had a small problem with their labeling machine and a few cans in the store owner’s lot must have been mislabeled before they caught the problem. The manufacturer offered to replace all 100 cans from the original order. The store owner refused the offer and told the manufacturer not to deliver the second lot, because he could no longer trust the manufacturer. The owner was very sensitive to the hardware store’s good reputation, which he felt was harmed by this incident.

If the manufacturer brings a claim of breach regarding the second shipment which was due on June 30, how will the court likely rule?

A

The buyer did not have the right to cancel the second shipment, because the defects in the first shipment did not substantially impair the value of the entire contract.

53
Q

A plaintiff sued a defendant for defamation, asserting in her complaint that the defendant had called the plaintiff a thief in front of a number of business associates. The plaintiff calls two witnesses to the stand, both of whom testify that they heard the defendant refer to the plaintiff as a thief in front of the business associates. The plaintiff does not take the stand herself. The defendant pleads truth of the statement as an affirmative defense and calls a witness to the stand. The defense witness is prepared to testify that he was a co-worker of the plaintiff when the plaintiff supplemented her income by tending bar three nights a week. The witness will testify that he saw the plaintiff take a $20 bill from the tavern’s cash register and secrete the money in her pocket. The plaintiff’s attorney objects.

May the defense witness’s testimony be allowed?

A

Yes, as substantive evidence that the plaintiff is, in fact, a thief.

54
Q

A horse breeder offered to sell a colt to his neighbor and they agreed on a purchase price. The horse breeder subsequently received a letter from the neighbor thanking him for the sale and summarizing their agreement. The letter contained the neighbor’s alleged signature. When the horse breeder attempted to set up transfer of the colt, the neighbor denied that she agreed to purchase it. In a breach of contract action against the neighbor, the horse breeder offers into evidence the letter. The horse breeder testifies that he is familiar with the neighbor’s handwriting and recognizes the signature on the letter as being hers.

Assuming appropriate objection by the neighbor, who claims that she did not sign the letter, how should the trial court rule on the admissibility of the letter?

A

Admit the letter but instruct the jury that it is up to them to decide whether the letter is authentic.

55
Q

A husband and a wife were arrested by federal agents and charged with distributing obscene materials through the United States mails. When called before a grand jury, the wife refused to say anything, invoking her Fifth Amendment right to be protected from compelled self-incrimination. The husband was terrified of the grand jury and readily admitted under questioning that he sent obscene matter through the mail. He also incriminated his wife in the illegal activity. The thought of a trial and a prison term drove the husband over the edge, and he committed suicide two days before his trial was to begin. A month later, the wife was put on trial in federal district court. The federal prosecutor seeks to introduce a transcript of the husband’s grand jury testimony into evidence against the wife. The defense attorney objects.

How should the court rule on the admissibility of the grand jury transcript?

A

Inadmissible, because the husband’s testimony was not subject to cross-examination.

56
Q

A man and a woman who were fierce business competitors were both competing for a large job. The man submitted his bid and then went to the woman’s office and told her, “If you leave this office, I’m going to get you!” The woman merely laughed and said, “I’m about finished with my bid and will be leaving in a few minutes.” The man left the office but placed a large, heavy couch across the entrance to the woman’s office, hoping to keep her from leaving. Meanwhile, the woman finished the bid and tried to leave her office, but found that she could not open the door. She pushed against the door as hard as she could and was eventually able to force it open, then ran all the way to the place where bids were being taken and got her bid in with one minute to spare. As usual, her bid was slightly lower than the man’s, and she was awarded the contract.

If the woman sues the man, what causes of action can she assert?

A

False imprisonment, but not assault.

57
Q

A driver traveling the speed limit in the evening on a quiet country road rounded a curve and struck a bicyclist who was riding in the same lane. The driver stopped the car and inspected the bicyclist, who had a broken leg. The driver thought it best not to try to move the bicyclist, so he told him that he would go to get help. The driver drove away and left the bicyclist by the side of the road. After the driver had left the scene, he realized that he had forgotten his wife’s birthday, so he stopped to buy a gift and hurried home. He did not remember the bicyclist until a few hours later, but assumed that by that time someone would have come along to render assistance. However, the bicyclist was not rescued until the following morning. By then, he had contracted pneumonia as a result of exposure.

The bicyclist sued the driver to recover damages for his broken leg and the pneumonia.

If the jury finds that the driver was not negligent in his operation of his automobile, for what harm will the bicyclist most likely recover?

A

The pneumonia but not the leg injury.

58
Q

A pilot was flying her small airplane when she realized that she was rapidly losing fuel and would not make it to the nearest airport. Looking down, she could find no large open space on which to attempt a landing except for a highway off to her left and a nearby lake about a mile to her right. She considered ditching the plane in the water but decided against it under the circumstances. As the pilot maneuvered over the highway and saw a long section free of any overpass or obstruction, her engine sputtered and died. In a barely controlled glide, the pilot descended onto the highway, but her left wing sideswiped the median and her plane veered to the right, crashing into a car. The plane and car catapulted into a fence, severely injuring both the pilot and driver.

The driver brought an action for personal injuries against the pilot. At trial, the above facts were established, and the parties stipulated that the sudden loss of fuel was due to a defect in the fuel system that could not have been discovered by the pilot. At the close of the evidence, both parties moved for a directed verdict.

How should the court rule?

A

Deny both motions, because the jury could decide that the pilot’s selection of the highway rather than the lake was not a reasonable choice under the circumstances.

59
Q

A landowner owned several dozen acres of mountain land near a national forest. A plaintiff who was injured by a condition on the owner’s land brought an action for personal injury against the landowner.

In a jurisdiction that applies the traditional rules for landowners and possessors of land, which of the following plaintiffs is most likely to win?

A

A five-year-old trespasser who fell into a mineshaft from which the owner had removed all warning signs, but the plaintiff was not attracted onto the owner’s land because of the mineshaft.

60
Q

A tenant invited a friend over for dinner. On his arrival, the friend stepped on a split board on the front steps and the board broke, causing him to lose his balance and break his ankle.

If the friend sues the tenant for his injuries and does not prevail in a jurisdiction that applies the traditional rules for landowners and possessors of land, what is the most likely explanation?

A

The friend should have noticed the dangerous condition himself.

61
Q

A camper at a state park built a campfire within a fire ring on a calm day according to approved procedures. Just as a sudden strong wind arrived and blew some embers onto the grass, a large bear came out of the woods and charged at the camper. The camper ran to his car, which was some distance away, with the bear in close pursuit. By the time the bear left and the camper was able to exit the car and summon assistance, the embers in the grass had started a brush fire. The fire destroyed another camper’s equipment and automobile at a nearby campsite before it could be extinguished.

The other camper sued the camper who started the fire. At trial, the parties stipulated to the above facts. The plaintiff introduced into evidence a state statute that prohibited leaving any campfires unattended and required them to be extinguished immediately if any embers were blown out of the fire ring. At the conclusion of the proofs, both parties moved for a directed verdict.

How should the court rule on the motions?

A

Grant the defendant’s motion, because the plaintiff has not established a prima facie case of negligence.

62
Q

A tenant’s apartment was without hot water for over a week because of a broken water heater, even though the landlord had been notified right away and the lease provided that the landlord would make repairs promptly. The tenant heated a large pot of water on the stove and started to carry it to the bathroom so she could warm up her bath. Her young nephew, who was visiting for a few days, came around the corner suddenly and collided with her. The hot water spilled on the nephew, burning him. Because the nephew had a rare blood disorder, the burns resulted in several of the nephew’s toes requiring amputation. The nephew’s guardian brought a negligence action against the landlord in a jurisdiction that follows the traditional rules for landowner liability.
If the jury finds in favor of the landlord, what is the most likely reason?

A

The landlord’s conduct was not the proximate cause of the nephew’s injuries.

63
Q

The plaintiff was driving inattentively when she had to swerve to avoid two other negligently driven vehicles at a busy intersection, and her car struck a light pole. The plaintiff, who was the only driver injured, sued one of the other drivers to recover damages in a jurisdiction that has adopted pure comparative negligence. The jury determined that she suffered injuries of $100,000 and was 50% at fault.

If the plaintiff is awarded a recovery of only $25,000 from the defendant, what will be the most likely reason?

A

The jurisdiction has abolished joint and several liability.

64
Q

A driver on a 3,000-mile cross-country trip in his new car tried to drive the entire trip without stopping, but fell asleep at the wheel, causing the car to strike a bridge abutment and roll over. The driver was seriously injured by the rollover, and suffered additional injuries when the turn signal rod broke off and punctured his lung.

The driver had purchased the car from his local auto dealer. The car was manufactured by a local manufacturer, and the turn signal rod was manufactured by a subcontractor whom the manufacturer had used for many years. Tests after the accident established that the turn signal rod was defective and that the defect was the reason it broke off. The defect was not discoverable through reasonable inspection and the manufacturer had had no prior indication of any defects.

The driver brought a strict liability action against the manufacturer in a jurisdiction that does not apply its comparative negligence rules to strict liability actions.

What is the likely result of the driver’s action?

A

The driver will be awarded damages for injuries incurred because the turn signal rod was defective, but he will not recover for other injuries incurred in the accident.

65
Q

A homeowner from State A hired a contractor from State B to build a vacation home for her in State C. The parties signed the contract in State A. The contractor breached the contract, and the homeowner sued the contractor in a court of State A, seeking damages of $100,000. The contractor removed the case to the federal court for State A. The homeowner promptly moved to remand the case to state court, arguing that venue was improper.

Which of the following facts is most relevant to the court’s decision on the homeowner’s motion?

A

The homeowner commenced the action in a State A court.

When a case is removed from state court to federal court, venue is set in the federal district court that embraces the state court in which the action was pending, making the federal district court of State A the only appropriate venue choice. Thus, (C) is the correct answer because the most (indeed, only) relevant fact is the fact that the action was commenced in a court of State A

66
Q

A builder from State A sued a homeowner from State B for breach of contract in federal court, alleging that the homeowner failed to pay the second half of the agreed-upon price for completion of construction on a house. The process server attempted to serve the summons and complaint on the homeowner at the house, but after two failed attempts, instead served it on an elderly next-door neighbor, who was out in the front yard and volunteered to accept it on the homeowner’s behalf.

The homeowner then filed a motion to dismiss for insufficient process. Assume that the requirements for service of process in both states are identical to the requirements of the Federal Rules of Civil Procedure.

Is the court likely to dismiss the action for insufficient process?

A

No, because the homeowner filed the wrong motion.

The court will not likely dismiss the action. Insufficient process refers to defects in the documents and their content, whereas insufficient service of process refers to the manner in which the documents were presented, delivered and/or served to the defendant. The key to this answer is reading comprehension. The facts state that the homeowner filed a “motion to dismiss for insufficient process,” not “insufficient service of process,” which are two separate grounds for dismissal. [See Fed. R. Civ. P. 12(b)(4) and (5),

67
Q

Shortly before their wedding, a man and a woman bought a tract of land, taking title in both names. They had intended to build a summer cottage there, but many years after their marriage the land was still a vacant lot. The man decided that their introverted son would have more confidence if he were a landowner; thus, the man drew up a deed conveying a one-quarter interest in the land to him. Not wanting to show favoritism, two weeks later the man drew up a deed conveying a one-quarter interest in the same land to their daughter.

Who owns the land?

A

The son has a one-quarter interest, the daughter has a one-quarter interest, and the woman has a one-half interest.

The son has a one-quarter interest, the daughter has a one-quarter interest, and the woman has a one-half interest. There is a presumption that the man and woman are tenants in common. For a joint tenancy to exist, there must be an express creation of such tenancy; thus, there is a presumption of tenancy in common unless the conveyance is to a husband and wife in a state that recognizes tenancy by the entirety. Here, the man and woman were not married when they took title to the land. Each tenant in common has an undivided interest, which may be conveyed by inter vivos transfer. The man started with an undivided one-half interest, one-half of which he conveyed to his son and the other half of which he conveyed to his daughter. The man has thus conveyed all of his interest in the land,

68
Q

A pedestrian brought suit against the driver of a car that struck him and caused him serious injuries and amnesia. During case preparation, the pedestrian’s attorney was frustrated by his client’s inability to remember any of the facts surrounding the accident. He therefore hired a qualified hypnotist to question the pedestrian. While hypnotized, the pedestrian described details of the accident, stating that the car that struck him ran a red light, and the car’s license plate number was XYZ356. It so happens that XYZ356 is the license number of the driver’s vehicle. At trial, the driver stipulated that his car struck the pedestrian, but asserts a defense that he was not negligent and that the light was green when he entered the intersection. Because the pedestrian’s amnesia persists at the time of trial, the pedestrian’s attorney wishes to call the hypnotist to the stand to testify as to the statements made by the pedestrian under hypnosis. The driver’s attorney objects.

How should the court rule on the objection?

A

Sustained, because the hypnotist’s testimony would be hearsay not within any recognized exception to the hearsay rule.

The hypnotist’s proposed testimony is hearsay evidence. It would be recounting an out-of-court statement (the pedestrian’s hypnotically induced recollection of the accident) for the purpose of establishing the truth of the matter asserted in the statement. Therefore, the issue in deciding upon the admissibility of the hypnotist’s testimony is to ascertain whether it fits within a hearsay exception. There is no hearsay exception for out-of-court statements given under hypnosis. The only hearsay exception that seems possibly applicable is the residual or “catch-all” exception. Before evidence can be admitted under this exception, however, an elaborate pretrial notice procedure must be followed to assure that the other party has an opportunity to prepare to meet the hearsay evidence. No such notice was given in this case

69
Q

MBE Practice Questions Set 26
7 of 147 of 14 Items

Question
Due to budget shortages and a critical need of funding to fight a war, Congress enacted a $25 tax on each person flying into an airport in the five most popular vacation destinations in the country, as determined by Congress. The tax was implemented, and officials in the five destinations were outraged, fearing that the number of vacationers to the taxed destinations would decrease due to the tax.

If the tax is challenged in federal court by an official with standing, is the most likely result that the tax will be held constitutional?

A

Yes, because Congress has plenary power to impose taxes to raise revenue.

The destination tax will likely be held constitutional under Congress’s taxing power. Congress has the power to lay taxes under Article I, Section 8, and a tax measure will usually be upheld if it bears some reasonable relationship to revenue production or if Congress has the power to regulate the taxed activity. Despite the protest from the officials of the affected locations, the tax here does appear to be related to revenue production and so will be upheld.

70
Q

A bartender diligently followed the procedure her employer set: She would ask every patron for identification regardless of how old (or young) the patron appeared to be. One day, after asking for identification, the bartender served alcohol to a minor. The identification that the minor gave to the bartender was actually issued by mistake by an appropriate state agency and appeared to show that the minor was of legal age. After another patron, an off-duty police officer, recognized the minor, the bartender was arrested for serving alcohol to a minor. In this jurisdiction, the highest state court has held that, under state law, strict liability is abolished and all crimes require a culpable mental state.

What would be the best reason for finding the bartender not guilty?

A

She did not know that the minor was underage, and she relied on the identification card for proof of age.

An honest and reasonable mistake as to a material element of the offense would negate criminal liability for all crimes except strict liability offenses. Thus, if the state had abolished strict liability crimes, the bartender’s mistake would be a defense regardless of the mental state required for the crime of serving alcohol to a minor

71
Q

The owner of a television agreed to sell it to a neighbor for $250. The neighbor made a down payment of $70, took possession of the television and agreed to pay the outstanding balance in nine equal $20 installments, beginning on June 5, with subsequent installments due on the fifth of each month until the balance was paid in full.

The neighbor’s friend owed her $200. On May 20, the neighbor and her friend entered into an oral agreement whereby the friend agreed to make the nine $20 installment payments to the seller in exchange for the neighbor’s promise to forgive the friend’s $200 debt. On June 7, the seller called the neighbor to ask her where his first $20 installment payment was, and she told him at that time of her agreement with her friend. The friend has made none of the installment payments.

If the seller files suit against the friend demanding payment, who will prevail?

A

The seller, because he was a third-party beneficiary of the agreement between the neighbor and her friend.

The neighbor has delegated her duties under the agreement with the seller to her friend, and the friend has agreed to assume the duties by agreeing to make the installment payments to the seller. Where a delegate’s promise to perform the delegated duty is supported by consideration, there results a third-party beneficiary situation, so that the nondelegating party to the contract can compel performance or bring suit for nonperformance. The friend’s promise to make the payments to the seller, totaling $180, was given in exchange for the neighbor’s promise to forgive the $200 debt owed by the friend to her. The neighbor thus relinquished her right to take action against her friend for the full amount owed, thereby incurring legal detriment. Consequently, the promise of the friend was supported by consideration, and a situation arose in which the seller became a third-party beneficiary of the agreement between the neighbor and her friend, and able to enforce performance of the friend’s promise to pay.

72
Q

An investor who owned several thriving shopping malls was negotiating to purchase a local mall from the company that currently owned it. A staff attorney for the state transportation department who shopped at the mall regularly learned of the negotiations and contacted the investor. The mall had deteriorated noticeably during the time the current company had owned it and the attorney believed that new ownership would revitalize the mall considerably. Although the attorney had no information to support this, she told the investor that the state was currently planning to construct a new interchange for the turnpike only three blocks from the mall. The investor went ahead with the purchase, believing that the new interchange would boost sales. In fact, no interchange was being considered by the state at that time, and nothing that the investor did after he purchased the mall could stem the decline in sales. He ended up selling the property at a substantial loss several years after the purchase.

Does the investor have a cause of action against the attorney for his losses?

A

Yes, for intentional misrepresentation, because the attorney was aware that she did not know whether the state was planning an interchange.

The attorney acted with scienter for purposes of an intentional misrepresentation action because she was aware that she did not know whether the state was planning an interchange. To establish a prima facie case of intentional misrepresentation or fraud, plaintiff must prove (i) misrepresentation by defendant, (ii) scienter, (iii) intent to induce plaintiff’s reliance on the misrepresentation, (iv) causation (actual reliance on the misrepresentation), (v) justifiable reliance on the misrepresentation, and (vi) damages. The element of scienter, which involves defendant’s state of mind, requires plaintiff to show that defendant made the statement knowing it to be false or made it with reckless disregard as to its truth or falsity. Because the attorney made her statement even though she had no information that the state was planning an interchange, she acted with scienter. The other elements of intentional misrepresentation are established by the facts.

73
Q

A State A citizen and a State B citizen were in a car accident in State A. The State A citizen filed a negligence action in a State A state court seeking $200,000 in damages. The State B citizen filed an answer and four months later filed a notice of removal, removing the action to federal court. Two months after the State B citizen filed and served the notice of removal, the State A citizen filed in the federal court a motion to remand the case back to state court.

Should the federal court remand the action to state court?

A

No, because the motion to remand the action to state court was untimely.

The federal court should not remand the action to state court. A defendant may remove an action that could have originally been brought in the federal courts. Generally, a defendant must file a notice of removal within 30 days after receipt by or service on the defendant of the initial pleading or summons (when he learns the case is removable). A defendant may not remove a case based solely on diversity if he is a citizen of the forum state; however, that restriction is not jurisdictional but rather procedural. Once removed, a case must be remanded back to state court if the federal court lacks subject matter jurisdiction. A motion to remand based on procedural defects in removal must be filed within 30 days of removal. Here, the federal court has diversity of citizenship jurisdiction over the action. The plaintiff is from State A and the defendant is from State B (and thus not a citizen of the forum state), and the amount in controversy is $200,000. Although the notice of removal was untimely, the motion to remand was also untimely. A motion to remand on grounds other than a lack of subject matter jurisdiction (e.g., the notice of removal was untimely filed) must be filed within 30 days of removal. The failure to timely remand the case on procedural defects is fatal.

74
Q

An entrepreneur opened a specialized business on her land. After using up most of her capital to purchase inventory, however, the entrepreneur needed more funds and asked her friend for a $30,000 loan, to be secured by the business’s inventory. The friend declined the loan. A desperate entrepreneur then told the friend she would convey the land, which had a fair market value of $100,000, to him if he would give her the loan at the current market rate of interest. The friend agreed, and the entrepreneur conveyed the land to the friend the next day. At that time, the friend gave the entrepreneur $30,000 in cash, and the parties orally agreed that the entrepreneur would pay the friend back at the rate of $1,000 per month, and that after the loan was paid in full, the friend would reconvey the land to the entrepreneur. The friend immediately recorded his deed to the land.

The entrepreneur made three $1,000 payments to the friend and then paid no more. She continued to live on the land but, being very much in debt, could not repay the loan. The friend, meanwhile, had received an offer to buy the land for $100,000.

Which of the following most accurately states the friend’s right to sell the property?

A

The friend may sell the land only after formally foreclosing on the property.

75
Q

A merchant sued a company for breach of contract, alleging that the products she purchased failed to conform to contract specifications. Shortly before the trial was to begin, the merchant suffered a stroke that left her paralyzed and virtually unable to communicate. Her guardian was properly substituted as the plaintiff in the lawsuit. At trial, following presentation of the plaintiff’s case, the company calls as a witness a priest to question him about a conversation he had with the merchant at a church fundraiser. In this conversation, the merchant told the priest in confidence that the products she received were actually quite functional, but that she had become aware of a lower price being offered by another vendor, and thus wanted to get out of her contract with the company. The plaintiff’s attorney immediately objects on the basis of clergy-penitent privilege

A

Overruled, because the circumstances under which the merchant made the statement take it outside the scope of the privilege.

The plaintiff’s objection on the basis of the clergy-penitent privilege should be overruled. Pursuant to the clergy-penitent privilege, a person has a privilege to refuse to disclose, and to prevent others from disclosing, a confidential communication by that person to a member of the clergy in the clergy member’s capacity as a spiritual adviser. The operation of this privilege is very similar to that of the attorney-client privilege. Here, the merchant made the statement to the priest during a conversation at a social occasion. There is no indication that this was a communication made to the priest in his capacity as a spiritual adviser, as would be the case, for instance, with a statement made in the confessional or during a counseling session. Thus, the matters stated to the priest by the merchant do not come within the clergy-penitent privilege, and the priest cannot be prevented from disclosing the contents of the conversation on the basis of this privilege.

76
Q

At a products liability trial, a critical issue is whether the temperature was below freezing on January 16. A local man who works for a civil engineering firm is also an avid amateur meteorologist. One of the man’s weather detection instruments in his backyard records temperature by markings from a stylus on a round barograph. The man’s record of the day in question indicates that it was unseasonably warm and that the temperature never fell lower than 48 degrees Fahrenheit, 16 degrees above the freezing mark. The plaintiff offers into evidence the man’s barograph record of the temperature on January 16.

Is the barograph record admissible?

A

No, unless foundation testimony is given as to the accuracy and good working order of the man’s instrument on the day in question.

77
Q

A mother and father instructed their son who just turned age 14 to report to a community woodworking shop instead of school. A state law requires all children to attend school until the age of 16, and the woodshop does not qualify as a school under state law. Because the parents did not report their son’s absence, a truant officer visited the family and warned them that parents who willfully refuse to comply with the mandatory attendance law are subject to a $500 fine and up to 30 days in jail for each day of noncompliance. The parents listened, but informed the officer that they could not comply with the state law because of their religious views, under which woodworking is an essential spiritual pursuit. The following day, the 14-year-old again went to work in the community woodshop instead of to school. His parents were then arrested and charged with violating the state mandatory school attendance law.

At the parents’ criminal trial, which of the following may the court constitutionally consider in determining whether First Amendment protection applies to the parents’ views?

A

Whether the parents’ belief system occupies a place in their lives similar to that occupied by orthodox religious beliefs.

Although the validity of religious beliefs cannot be questioned, the role a belief system plays in a person’s life can be examined to determine whether the beliefs are indeed religious. The Free Exercise Clause of the First Amendment, applicable to the states through the Fourteenth Amendment, prohibits punishing people for their religious beliefs. When a person claims that he is being punished for his religious beliefs, the court may consider whether the person challenging the law sincerely holds those beliefs. Thus, the court may consider whether the parents’ beliefs play a role in their lives similar to that of orthodox religious beliefs.

78
Q

While at a party, a wife came up behind a younger woman, grabbed her by her arm, and accused her of having an affair with the wife’s husband. The wife knew that her accusation was not true.

Of the following facts, which would be most helpful to the younger woman in a suit against the wife for intentional infliction of emotional distress?

A

The wife knew that the younger woman is very religious, and her religious beliefs strongly condemn adultery.

79
Q

In the course of repainting an apartment, the landlord of a small apartment building used a professional strength, stain-killing primer manufactured by a paint company for professional painters. The building’s common ventilation system was running as the landlord applied the primer, and some fumes from the primer went through the ventilation system into the apartment of the upstairs tenant, who suffered injuries to her eyes as a result. The warning label on the can, which the landlord read, stated: “Danger. This material is extremely hazardous and volatile. Do not use near open flame. Use only with adequate ventilation.” The product contained a chemical known to be harmful to people’s eyes, but in the 15 years that the product has been on the market, there were no reported cases of anyone suffering an eye injury from the product. However, professional painters routinely close off or shut down any common ventilation systems in buildings before using the product.

If the tenant brings an action against the paint company on a theory of strict liability, will she recover

A

Yes, because the label on the product did not warn of the risk of the fumes causing eye injury.

80
Q

A statute passed by both houses of Congress and signed by the President authorizes a federal agency to select a site for and to construct a monument honoring members of the capitol police force killed in the line of duty. The statute appropriates the necessary funds but provides that the funds may not be expended until both houses of Congress have adopted a concurrent resolution, not subject to presentment to the President, approving the agency’s plans for the monument’s location and design.

Is the provision requiring further congressional approval before expenditure of the funds constitutional?

A

No, because the provision amounts to an unconstitutional legislative interference with an executive function.

81
Q

A man was in jail after being arrested for burglary. When the police attempted to question him, the man invoked his Miranda rights and refused to answer any questions. The man was subsequently tried, convicted, and sentenced to a prison term for the burglary. Three years later, while the man was serving his prison sentence for the burglary, a police detective from a nearby town questioned him about an unsolved homicide. The detective did not know that the man had invoked Miranda at an earlier time. The man waived his Miranda rights and made several incriminating statements to the detective. When he was later charged with the homicide, the man moved to suppress these statements, claiming that his earlier refusal to waive his Miranda rights should have been honored.

Should the court suppress the statements?

A

No, because the man’s prior invocation of his Miranda rights did not preclude the later interrogation.

As a general rule, police officers must scrupulously honor a detainee’s invocation of Miranda rights. If the invocation is not scrupulously honored, statements made by the detainee are inadmissible. The prohibition against questioning a detainee after invoking Miranda rights lasts the entire time the detainee is in custody for interrogation purposes, plus 14 more days after the detainee returns to his normal life (which can include his “normal life” in jail). Here, the detainee invoked Miranda rights three years before the questioning in the current case. Thus, far more than 14 days had passed since he had returned to his “normal life” in jail.

82
Q

A defendant is on trial in federal court for bank robbery. Before the police had any suspects, a police officer interviewed an eyewitness at the police station and showed her a “mug book” containing dozens of photographs. The eyewitness identified the defendant’s photograph as that of the robber.

At trial, the eyewitness surprises the prosecutor by testifying that she is unable to identify the defendant as the robber. The prosecutor calls the officer to testify that the eyewitness identified the defendant from the photograph in the police station. The eyewitness remains present in the courthouse and can be recalled.

Is the officer’s testimony admissible?

A

Yes, because the eyewitness’s statement of identification as reported by the officer is not excluded by the hearsay rule.

A declarant’s prior statement identifying someone she perceived earlier is not hearsay under the Federal Rules of Evidence if the declarant is now testifying at trial and subject to cross-examination about the prior statement. Here, the eyewitness testified at trial and can be recalled and cross-examined, so her prior statement identifying the defendant as the robber is admissible.

83
Q

A general contractor about to bid on a construction job for an office building invited a carpenter and several others to bid on the carpentry work. The carpenter agreed to bid if the general contractor would agree to give the carpenter the job provided that his bid was lowest and the general contractor was awarded the main contract. The general contractor so agreed. The carpenter, incurring time and expense in preparing his bid, submitted the lowest carpentry bid. The general contractor used the carpenter’s bid in calculating its own bid, which was successful.

Which of the following best supports the carpenter’s position that the general contractor is obligated to award the carpentry subcontract to the carpenter?

A

The carpenter gave consideration for the general contractor’s conditional promise to award the carpentry subcontract to the carpenter.

The carpenter’s bid was consideration for the general contractor’s promise to award the carpentry subcontract to the carpenter if his bid was the lowest and the general contractor was awarded the main contract. Thus, the general contractor and the carpenter formed a contract.

84
Q

At trial in an action for personal injuries suffered in a traffic accident, the plaintiff first calls the defendant as an adverse party. The plaintiff then calls a witness who was a passenger in the plaintiff’s car but who also happens to be the defendant’s former employer. On direct examination, the witness testifies to how the accident occurred and also expresses his opinion that the defendant is not a truthful person.

Which one of the following areas of questioning is most likely to be held beyond the proper scope of cross-examination?

A

The plaintiff’s injuries were not as serious as the plaintiff is claiming.

The scope of cross-examination is generally limited to: (i) matters brought up on direct examination; and (ii) matters concerning the witness’s credibility (i.e., impeachment). Here, the witness’s testimony on direct examination concerned how the accident happened and the defendant’s character for untruthfulness; there is no indication that the witness testified about the plaintiff’s injuries. Because the questioning does not concern the subject matter of the direct examination or the witness’s credibility, it is most likely to be held beyond the proper scope of cross-examination. The other answer choices describe proper impeachment of the witness.

85
Q

A plaintiff sued the manufacturer of his lawn mower, alleging that, as the result of a design defect, he was injured when the mower’s blade flew off after striking a sprinkler head in the lawn.

At trial, the manufacturer has called as an expert witness a product safety engineer, who testifies that the manufacturer retained him for a fee to test identical mowers and, if his opinion was helpful, to testify in the case. The expert then testifies that he did test the mowers, and that the blade, as designed and installed by the manufacturer, could not fly off in the manner claimed by the plaintiff. Assume that the expert has used a reliable method for reaching his conclusion.

Should the court admit the expert’s testimony?

A

Yes, because expert testimony on such issues of causation is relevant and helpful to the jury.

This is the best answer choice because the other answers are clearly wrong. Expert testimony is admissible if: (i) the subject matter is one where scientific, technical, or other specialized knowledge would help the trier of fact understand the evidence or determine a fact in issue; (ii) the opinion is based on sufficient facts or data; (iii) the opinion is the product of reliable principles and methods; and (iv) the expert has reliably applied the principles and methods to the facts of the case. Here, the expert’s testimony as to his findings is relevant and helpful to the jury in determining whether the blade flew off in the manner claimed by the plaintiff, and the expert has used a reliable method in reaching his conclusion..

86
Q

A shop owner domiciled in State A sued a distributor in a federal district court in State A for breach of a contract. The shop owner sought $100,000 in damages for allegedly defective goods that the distributor had provided under the contract. The distributor is incorporated in State B, with its principal place of business in State C. The distributor brought in as a third-party defendant the wholesaler that had provided the goods to the distributor, alleging that the wholesaler had a duty to indemnify the distributor for any damages recovered by the shop owner. The wholesaler is incorporated in State B, with its principal place of business in State A.

The wholesaler has asserted a $60,000 counterclaim against the distributor for payment for the goods at issue, and the distributor has moved to dismiss the counterclaim for lack of subject-matter jurisdiction.

Should the motion to dismiss be granted?

A

No, because there is supplemental jurisdiction over the wholesaler’s counterclaim.

There is supplemental jurisdiction because the claim for nonpayment for the goods in issue arises out of a common nucleus of operative fact as the plaintiff’s (the shop owner’s) claims, and the restrictions contained in the supplemental jurisdiction statute do not apply to these facts.

87
Q

A state generally provides funding for the medical care of its residents who cannot afford such care. State law, however, prohibits use of this state funding for surgery for any person who has resided in the state for less than one year, except in emergency situations.

A woman moved to the state two months ago seeking permanent employment. Her physician recommends non-emergency surgery to treat a medical condition. The surgery would qualify for state funding if the woman had resided in the state for a year. The woman has sued to invalidate the state law that prohibits state funding of her surgery.

Should the woman prevail in her action?

A

Yes, because the law burdens the woman’s fundamental right to travel.

The right to travel is protected by the Fourteenth Amendment, and it includes the right of newly arrived citizens to enjoy the same privileges as are enjoyed by other citizens of the state. The law here gives a benefit to long-term residents (people who have been residents more than a year) and so infringes on the right to travel.

88
Q

A man decided to give a cabin he owned to his daughter at his death. To accomplish this goal, he delivered to his attorney a deed that fully complied with the applicable statute of frauds and told his attorney to record the deed when he died unless he later gave the attorney instructions to the contrary.
Three weeks after dropping off the deed, the man properly drafted and executed his own will, which left all of his real property to his son.

One year later, the man died, and the attorney immediately recorded the deed. At the time of the man’s death, the cabin was titled in his name and he owned no other real property. The daughter and the son now disagree as to who is entitled to ownership of the cabin.

Other than the jurisdiction’s statute of frauds and statute of wills, there are no applicable statutes.

Who is entitled to ownership of the cabin?

A

The son, because the deed was not delivered to the daughter during the man’s lifetime.

89
Q

A plaintiff sued an industrial facility in her neighborhood for injuries to her health caused by air pollution. At trial, the plaintiff was asked questions on direct examination about the days on which she had observed large amounts of dust in the air and how long the condition had lasted. She testified that she could not remember the specific times, but that she maintained a diary in which she had accurately recorded this information on a daily basis. When her attorney sought to refresh her recollection with her diary, she still could not remember. The plaintiff’s attorney seeks to have the information in the diary admitted at trial.

Is the information admissible?

A

Yes, and the plaintiff should be allowed to read the diary into evidence.

Where a witness states that she has insufficient recollection of an event to enable her to testify fully and accurately, even after she has consulted a memorandum or other record given to her on the stand, the record itself may be read into evidence if a proper foundation is laid for its admissibility. This is known as the recorded recollection exception to the hearsay rule. Here, all the foundational requirements are met-the plaintiff had personal knowledge of the facts when she made the diary; she made the diary herself and in a timely manner each day; she has vouched for its accuracy; and she currently has insufficient recollection to testify fully and accurately about the facts contained in the diary. Thus, the information in the diary may be read into evidence.

90
Q

An individual investor purchased stock through a company’s stock offering. When the price of the stock plummeted, the investor sued the company in a state court in State A, claiming that the company’s offering materials had fraudulently induced him to purchase the stock and seeking $25,000 in damages.

A university that had purchased the company’s stock through the same offering sued the company in federal court in State B, claiming that the offering materials violated federal securities laws and seeking $1 million in damages.

The individual investor’s suit proceeded to trial. The state court ruled that the company’s offering materials contained false information and awarded the investor a $25,000 judgment. The university immediately moved for partial summary judgment in its federal action against the company, arguing that the state court judgment bound the federal court on the issue of whether the company’s offering materials contained false information.

Neither State A nor State B permits nonmutual issue preclusion.

Should the court grant the university’s motion?

A

No, because State A does not permit nonmutual issue preclusion.

91
Q

A plaintiff sued the insurer of her home after the insurer denied coverage for water damage to the home allegedly caused by a frozen plastic pipe that burst. At trial, the insurer called as an expert witness an engineer, who testified that the pipe had burst because of age rather than freezing. On cross-examination, the engineer admitted that, five years earlier, he had been convicted of tax fraud, even though he had asserted that it was his accountant’s error. In response, the insurer calls a witness, who is well acquainted with the engineer and his reputation, to testify that (1) in the witness’s opinion, the engineer is a truthful person, and (2) the engineer’s neighbors all describe him as a truthful person.

How much, if any, of the witness’s testimony is admissible?

A

All of the testimony is admissible to support the engineer’s credibility.

92
Q

On the basis of scientific studies showing a causal relationship between the consumption of “red meat” (principally beef) and certain forms of cancer, a federal statute prohibits all commercial advertising of red meat products. The statute does not, however, restrict the sale of red meat products. Producers of red meat have challenged the statute as a violation of their free speech rights protected by the First Amendment.

Is the court likely to find the statute constitutional?

A

No, because it is more extensive than necessary to serve the government interest in preventing certain cancers

93
Q

A defendant was charged with and tried for murder. At trial, the defendant testified that he had acted in self-defense.

The trial court instructed the jury that the defendant had the burden of proving by a preponderance of the evidence that he had acted in self-defense. The defendant objected, arguing that this instruction violated the constitutional requirement that the prosecution prove the offense beyond a reasonable doubt.

Did the trial court’s instruction violate the defendant’s federal constitutional rights?

A

No, because due process does not preclude requiring a defendant to prove self-defense by a preponderance of the evidence.

In all criminal trials, the government has the burden to prove all the elements of a crime beyond a reasonable doubt. However, this rule does not preclude the state from imposing a burden on the defendant to prove an affirmative defense, such as self-defense.

94
Q

A pedestrian was injured when hit by a chair that was thrown from an upper-story hotel window. The pedestrian sued the occupants of all the rooms from which the chair might have been thrown.

At trial, the pedestrian has been unable to offer any evidence as to the exact room from which the chair was thrown. The defendants have filed a motion for a directed verdict.

Should the court grant the motion

A

Yes, because the pedestrian has failed to offer evidence that the defendants jointly engaged in tortious conduct.

95
Q

A plaintiff, a management trainee, brought a sex discrimination lawsuit against her employer for wrongful termination of her employment. At trial, the plaintiff is prepared to testify that a janitor at the company told her that he had heard her supervisor say to other male co-workers about her, “Make it hard for her. Maybe she’ll go home where she belongs.”

Is the plaintiff’s proposed testimony admissible?

A

No, because the janitor’s statement is hearsay not within any exception.

96
Q

A police officer was employed on a city’s police force for 10 years. When the officer accepted the job, the city’s employee benefit plan provided a death benefit to the spouse of any employee who died as a result of any job-related injury. Last year, the city amended its employee benefit plan to deny its death benefit in cases where the death “was caused by the employee’s refusal to accept, for any reason, reasonably available medical care prescribed by a physician.”

After this amendment took effect, the officer was shot while on duty. Because of a sincerely held religious belief, the officer refused to allow a prescribed blood transfusion and, as a result, died from loss of blood. When the officer’s spouse applied for the death benefit, the city denied the application on the basis of the amendment to the employee benefit plan.

The officer’s spouse has challenged the amendment, claiming that, as applied to the officer, it violated the officer’s constitutional right to the free exercise of religion.

Is the court likely to find the amendment to the employee benefit plan constitutional as applied to the officer?

A

Yes, because it is generally applicable and religiously neutral, as well as a reasonable limitation on the award of such benefits.

97
Q

A construction contractor brought a breach of contract claim in federal court against a homeowner who had hired the contractor to build an apartment over an existing garage. The action turned on the scope of the work covered by the contract. The contractor and the homeowner were the only witnesses at the bench trial, and they strongly disagreed about the scope of the work. At the end of the trial, the judge stated findings of fact on the record but never issued a written opinion. Neither party objected to the findings. The judge found in favor of the homeowner, and the contractor appealed.

Is the appellate court likely to overturn the findings?

A

No, because the appellate court must give due regard to the trial judge’s opportunity to determine witness credibility.

98
Q

A man became ill while at work and decided to go home early. When he entered his bedroom, the man saw his wife engaged in sexual intercourse with a neighbor. The man grabbed a gun from a dresser drawer and shot and killed the neighbor. He was later charged and prosecuted.

In a jurisdiction that follows the common law for homicide offenses, which crimes should the court instruct the jury on?

A

Murder and voluntary manslaughter.

99
Q

A patient domiciled in State A sued a surgeon domiciled in State B in a federal court in State A, alleging claims for malpractice. The surgeon moved to dismiss the action for lack of personal jurisdiction. The court denied the motion and set discovery cutoff and trial dates.

The surgeon has appealed the denial of the motion.

Should the appellate court hear the merits of the surgeon’s appeal?

A

No, because the appellate court lacks jurisdiction over the appeal.

only final judgments may be appealed.

100
Q

Ten years ago, a couple bought a building and moved into its second-floor apartment with their teenage daughter. The couple operated a shoe store on the first floor of the building for many years. When the couple purchased the building, the area was predominantly rural and was zoned for nonresidential use. The municipality’s zoning is cumulative.
Five years ago, the municipality rezoned the area to single-family residential use. The daughter was not aware of this change, since she was away at college.

Recently, the daughter inherited the building from her parents. The daughter immediately moved into the apartment and took over the operation of the shoe store on the first floor. The daughter has learned that a developer is planning to build a large residential community in the area surrounding her building.

The daughter has asked her lawyer for advice regarding her ability to continue operating the shoe store.

Should the lawyer advise the daughter that she can continue to operate her shoe store?

A

Yes, because the shoe store is a nonconforming use.

However, a use that exists at the time of passage of a zoning ordinance and that does not conform cannot be eliminated at once. Generally, the nonconforming use may continue indefinitely, but any change in the use must comply with the zoning ordinance. Because the shoe store existed at the time of the rezoning, the daughter may continue to operate the shoe store as a nonconforming use.

101
Q

A defendant has been charged with making a false statement to a federally insured financial institution to secure a loan. At trial, the prosecutor calls the defendant’s wife as a willing witness to testify that the defendant told her in confidence that he had misrepresented his assets on the loan application. The defendant objects to his wife’s testimony.

Should the testimony be admitted?

A

No, because even though the wife is a willing witness, the defendant has the right to exclude confidential marital communications in federal court.

102
Q

A city filed eminent domain proceedings in order to obtain 40 beach houses fronting a particularly attractive stretch of shoreline. As part of an elaborate plan to increase the city’s tourist trade and revive the local economy, the city planned to sell the beach houses to a company that would demolish the houses and build a luxury hotel in their place.

The owners of the beach houses have challenged the city’s exercise of eminent domain, contending only that the city’s plan is unconstitutional.

Will the owners of the beach houses be likely to prevail?

A

No, because the planned sale to the private developer to increase the tourist trade qualifies as a public use.

The Fifth Amendment prohibits taking of private property for public use without just compensation. The Supreme Court has held that taking property to turn it over to a private developer can still qualify as a public use.